Вы находитесь на странице: 1из 13

Inequality Problems in QQAD so far

Type: Manipulating Expressions


Which of the following statements about the functions
A(x, y, z) = x + 3y - 4xy - 2yz + 2zx and B(x,y,z) = |x-|x-y|| + |y-|y-z|| + |z-|zx|| is necessarily true?
(a) B(x,y,z) <= |x-y| + |y-z| + |z-x|
(c) A(x,y,z) > -6

(b) (B(x,y,z))^2 > A(x,y,z) for x > y > z > 0

(d) None of the above

Since all the terms in B(x,y,z) are symmetric we assume without the loss of any
generality that x >= y >= z
Thus, B(x,y,z) = |x-|x-y|| + |y-|y-z|| + |z-|z-x|| = |y| + |z| + |2z-x|
|x-y| + |y-z| + |z-x| = x-y + y-z + x-z = 2(x-z)
putting x = 1, y=1, z= 1 we see that choice (a) is not true
x + 3y - 4xy - 2yz + 2zx = (x - 2y + z)^2 -y^2 -z^2 + 2yz
=> A(x,y,z) = (x - 2y + z)^2 -(y-z)^2
=> A(x,y,z) = (x - 3y + 2z)(x-y)
Let x > y > z > 0 and x > 2z, then B(x,y,z) = x+y-z, if (B(x,y,z))^2 > A(x,y,z) =>
(B(x,y,z))^2 - A(x,y,z) > 0, solving we get 2y(3x-y) + 2x(3y-2z) which is > 0
Let x > y > z > 0 and x < 2z, then B(x,y,z) = y+z+(2z-x) = 3z +y -x if (B(x,y,z))^2
> A(x,y,z) => (B(x,y,z))^2 - A(x,y,z) > 0, solving we get (3z +y -x)^2 - (x - 3y +
2z)(x-y) > 0 => 9z^2 - 2y^2 +8yz +2xy -8zx > 0
=> z(9z-8x+8y) + 2x(x-y) > 0, which we can see always is Hence, (b) is the right
option
put x=1, y = 0, z = -4 we get A(x, y, z) = -7
hence (c) is also not true

Type: Given expression is constant (source PGDCM papers of IIM C)


If x, y, z are any real numbers then the minimum possible value of x^2 + 2y^2 +
z^2 + 2yz subject of x + 2y + z = 6 is
(a) 6 (b) 6 (c) 12 (d) none of the foregoing
We are to find Min(x^2 + 2y^2 + z^2 + 2yz) = Min(x^2 + y^2 + (y+z)^2) subject
to x + 2y + z = 6
if x =a, y=b, y+z=c, then a^2 + b^2 + c^2 is to be minimized when a+b+c is
constant
Thus, a=b=c
x=y=y+z => x= -2, y= -2, z=0
Hence, Choice (c) is the correct answer
Type: Functional Inequality (source PGDCM papers of IIM C)
Let f(x,y) = 1 if |x - y| <= 1/2 = 0 otherwise
Then g(y) = f(1/4,y)f(3/4,y) is maximized with respect to y if only if
(a) y = 1/2 (b) y >= 3/4 (c) 3/8 <= y <= 5/8 (d) none of the foregoing
For g(y) to be maximum, f(1/4,y) and f(3/4,y) has to be maximum
when -1/4 <= y <= 3/4 f(1/4,y) = 1
when 1/4 <= y <= 5/4 f(3/4,y) = 1
=> y lies in [1/4, 3/4]
Hence, choice (d) is the correct answer.

Type: AM_GM rule (on non-negative numbers) and other


Which among the following 3 statements is/are true?
I. The minimum value of (x+5)(x+2)/(x+1) is 9 for x > -1
II. The maximum value of (100^n)/n! occurs at n = 100 only
III. For any real numbers x, y > 1, x^2/(y - 1) + y^2/(x - 1) >= 8
(a) Only II (b) Only III (c) Both I and II (d) none of the foregoing
I. Let (x+5)(x+2)/(x+1) = y = (x^2 + 7x + 10)/(x+1) = (x^2 + 2x + 1 + 5x + 5 +
4)/(x+1)
=> (x+5)(x+2)/(x+1) = x+1 + 4/(x+1) + 5
By AM-GM rule, x+1 + 4/(x+1) >= 4
=> I is true
II. (100^100)/100! = (100^99)/99!
For n > 100, (100^n)/n! = ((100^100)/100!)*(100^x/(101*102*...xterms)
where n = x + 100
But (100^x/(101*102*...xterms) < 1
For n < 100, (100^n)/n! < (100^100)/100! always as numerator/denominator
> 1 as n increases from 1 to 99
Hence, II is false
III. We have (x-2)^2 = 0, so x^2 = 4(x-1). Hence x/(x-1)^1/2 = 2.
Now by AM/GM, x^2/(y - 1) + y^2/(x - 1) = 2xy/((x-1)(y-1))^1/2.
But rhs = 222
=> III is true
Choice (d) is the answer

Type: Factorization (based on CAT 2003 question)


Let x = (n^4 + 256 + 4n(n^2 + 16))/(n+4)^2. If 4 <= n^2 <= 49 then,
(a) 12 <= x <= 95 (b) 28 <= x <= 95 (c) 12 <= x <= 37 (d) none of the
foregoing
(n^4 + 256 + 4n(n^2 + 16))/(n+4)^2 = (n^3 + 64)(n+4)/(n+4)^2 = n^2 - 4n +
16
4 <= n^2 <= 49 => -7 <= x <= -2 or 2 <= n <= 7
x = n^2 - 4n + 16 = (n-2)^2 + 12
x is max at n = -7, and min. at n=2
Hence, choice (d) is the right answer
Type: Basic Calculus
Let x be real and f(x) be the largest value of |y^2 - xy| for y in [0, 1]. What is the
minimum value of f (x)?
(a) 1/4 (b) 3 - 2*(2^1/2) (c) (2^1/2 - 1)/2 (d) none of the foregoing
y^2 - xy = (y - x/2)^2 - x^2/4, so y^2 - xy has a minimum at y = x/2. It is
decreasing for y < x/2 and increasing for y > x/2.
Thus the largest value of y^2 - xy must be at y = 0, x/2 or 1. The values there are
0, x^2/4, |1-x|.
So for x outside the interval (0,1), f(x) >= 1/4. For x in (0,1) we have f(x) =
max(x^2/4, 1-x). The quadratic x^2 + 4x - 4 = 0 has roots -2 2*2^1/2, which
are approx and -4.83 and 0.83. Hence f(x) = 1-x for x <= 2*2^1/2 - 2, and x^2/4
for x <= 2*2^1/2 - 2, and the minimum value of f(x) in the interval (0,1) is
f(2*2^1/2 - 2) = 3 - 2*2^1/2,
which is approx 0.18 and < 1/4. This is also the minimum value of f(x) for all x
Hence, choice (b) is the right answer
Type: Cauchy-Schwarz
Find the maximum value of 3x + 4y if x^2 + y^2 = 6x - 4y - 4
(a) 8

(b) 9

(c) 10

(d) none of the foregoing

(a^2 + b^2)*(c^2 + d^2) >= (ac + bd)^2


9*25 = ((x-3)^2 + (y+2)^2)*(3^2 + 4^2) >= (3(x-3) + 4(y+2))^2
=> 9*25 >= (3(x-3) + 4(y+2))^2
=> 3x + 4y <= 16
Hence, choice (d) is the correct answer
This was based on cauchy-schwarzs inequality.
Type: Cauchy-Schwarz
If a, b, c, d are real numbers with a^2 + b^2 + c^2 + d^2 = 100, then what is the
maximum value of 2a + 3b + 6c + 24d?
(a) 240 (b) 250 (c) 300 (d) none of the foregoing
Cauchy-Schwarz's inequality: (a1^2 + a2^2 + a3^3 +...)*(b1^2 + b2^2 + b3^3
+...) >= (a1*b1 + a2*b2 + a3*b3 + ...)^2
(a^2 + b^2 + c^2 + d^2)*(2^2 + 3^2 + 6^2 + 24^2) >= (2a + 3b + 6c + 24d)^2
100*625 >= (2a + 3b + 6c + 24d)^2
Hence, choice (b) is the right answer
Type: n^1/n assume max value at n = e = 2.718
The maximal possible value of the product of certain pairwise positive integers
whose sum is 100 is 4.(3^32)
Try yourself.
Type: A variant of above
The maximal possible value of the product of certain pairwise distinct positive
integers whose sum is 100 is
(a) 15!/48

(b) 14!/4

(c) 4*(3^32)

(d) none of the foregoing

We start of with 2*3*...*11*12*23 because we need to use all the smallest integers
because every integer greater than 3 can be replaced with a sum of two others
whose product is greater or equal than that integer .i.e. 4 = 2+2 <=2*2
5=

2+3 < 2*3 = 6 so this means we should just decrease the number 23 untill it gets
to 14 and increase all the other numbers by 1.
Then we get the solution 2*3*...*10*12*13*14 or shortly 14!/4
Hence, choice (b) is the right answer

Type: Calculus and Equations (QQAD 2006 and later appeared in XAT
2008)
If a, b, c are real numbers such that a < b < c and a + b + c = 6, ab + bc + ca = 9,
then which among the following is definitely true?
(a) 0 < a < 1 (b) 1 < b < 3 (c) 3 < c < 4 (d) All of them
Let t = abc. Then a, b, c are the roots (= zeros) of f(x) = x^3 - 6x^2 + 9x - t = 0.
We have f'(x) = 3(x - 1)(x - 3) so f(3) < 0 < f(1). Hence a < 1 < b < 3 < c.
Since f(1) = 4 - t > 0, t = abc < 4, and since f(3) = -t < 0, abc > 0.
As b, c are positive, a > 0.
Also, f(x) is strictly increasing on (3, +oo) and f(4) = f(1) = 4 - p > 0 so c < 4.
Hence, choice (d) is the right answer
Type: Another variant of Cauchy-Schwarz
The sum of five real numbers is 8 and the sum of their squares is 16. What is the
largest possible value for one of the numbers?
(a) 16/5

(b) 2

(c) 25/8

(d) none of these

a+b+c+d = 8-e, also a^2 + b^2 + c^2 + d^2 = 16 - e^2


(a^2 + b^2 + c^2 + d^2)(1+1+1+1) >= (a+b+c+d)^2
=> 4*(16 - e^2) >= (8-e)^2
=> e <= 16/5
Hence, choice (a) is the right answer.
Type: Inequality framed as geometry problem
The set of points x, y which satisfies x^3 - 3xy^2 >= 3x^2*y - y^3 and x+y = -1 is
a line segment whose length is
(a) 1/2

(b) (2/3)^1/2

(c) 3^1/2 - 1

(d) 1

x^3+y^3+3xy(x+y) = (x+y)^3
so given equations are
(x+y)^3-6xy(x+y) >=0, and x+y = -1
6xy>=1, and x+y = -1, y= -1-x
6.x(-1-x) >=1, 6x(1+x)<=-1
6x^2+6x+1<=0, so
x= (-3+sqrt3)/6 and (-3-sqrt3)/6, y is (2-sqrt3)/6 and (2+sqrt3)/6
distance will become 2sqt6/6 = sqrt(2/3)
Type: Squaring the expression
Let 0 <= x, y <= 1. The positive difference between the maximum and minimum
value of (x-2y+1)^2 + (x+y-1)^2 is
(a) 4 (b) 16/3 (c) 25/4 (d) none of the foregoing
The expression (x-2y+1)^2 + (x+y-1)^2 = 2*(x-1/2y)^2 + 1/2*(3y-2)^2. The
_expression becomes minimum when y = 2/3 and x= 1/2y = 1/3. The min value is
0.
The maximum value is achieved y = 0 and x = 1. The max value is 4.
Hence, choice (a) is the right answer.
Type: Breaking the expression and applying AM-GM
I) For positive p, q, r the minimum value of 2q/r + (1+r)/2p + 2p^2/q is 4
II) x^4 > x - 1/2 for all real x
Which among the above is true?
(a) Only I

(b) I && II

(c) Only II

(d) none of the foregoing

2q/r + (1+r)/2p + 2p^2/q = 2q/r + 1/2p + r/2p + 2p^2/q >= 4*(2q/r*1/2p*r/2p*


2p^2/q) >= 4 => I is true
x^4 - x + 1/2 = (x^2 - 1/2)^2 + (x - 1/2)^2 = 0. We could only have equality if
x^2 = x = 1/2, which is impossible, so the inequality is strict. => II is true
Hence, choice (b) is the right answer

Type: Word problem for AM_GM in equations


Apple had to study Mathematics from his teacher Orange. As usual, Apple was
sleeping in the class. Immediately he heard a thundering voice. "All roots of this
equation are real as well as positive in nature". Apple woke up from deep
slumber. He hurried to copy the 10 degree equation written on the board but
could copy only the first two terms written on the blackboard before Orange sir
wiped it all. Apple however remembered that the constant term was 2. He noted
down the equation as 2x^10 - 20x^9. + 2 = 0. Apple was very sure that if
someone would tell him the sum of all the coefficients of all the powers of x in the
equation, he would solve it anyhow. He asked Mango about the same. The answer
which Mango correctly gave was
(a) -1024 (b) 0
the foregoing

(c) 1024

(d) Mango himself was confused

(e) none of

Solution:
Let the roots be a1, a2,.., a10 then a1 + a2 + . + a10 = 10 , and (a1.a2.a10)=1
Now since all the roots are real and positive in nature
We can say that (a1+a2+.+a10)/10>=(a1.a2.a10)^1/10
For real numbers, AM >= GM but here we find that AM = GM hence a1 = a2 =
= a10 = 1
So our equation actually is 2(x-1)^10=0 and then the sum of all the coefficients of
all the powers of x in the equation is 0.
Hence, choice (b) is the right answer

Type: Pigeon Hole in Inequality


The minimum possible value of the largest of ab, 1-a-b+ab, and a+b-2ab if 0 <= a
<= b <=1 is
(a) 4/9

(b)1/9

(c)5/9

(d)1/3

(e) none of the foregoing

Solution:
Let s = a + b, p = ab, so a and b are (s+/- root(s^2-4p))/2 . Since a and b are
real , s^2 - 4p>=0 . If one of the three quantities is less than or equal to 1/9, then
at least one of the others is at least 4/9 by the pigeonhole principle since they add
up to 1. Assume that s-2p < 4=9, then s^2 - 4p < (4/9 + 2p)^2 - 4p , and since the

left side is non-negative we get 0<= p^2 -(5/9) p +4/81 =(p-1/9)(p-4/9).


This implies that either p<=1/9 or p>=4/9 , and either way we're done. This
minimum is achieved if a and b are both 1/3, so the answer is 4/9
Hence, choice (a) is the right answer

Type: Another AM-GM use for solving Inequality Problem


Let a, b, c be positive reals. (I) and (II) are independent statements.
(I) Minimum value of a^3/4b + b/8c^2 + (1+c)/2a is p
(II) a + b + 2c = 8 and a^2 + b^2 + 2c^2 = 25. Maximum possible value of c is q.
Then which among the following is |p-q|?
(a) 5/2

(b) 9/4

(c) 1/2

(d) 3/8

(e) none of the foregoing

Solution:
(I) a^3/4b + b/8c^2 + (1+c)/2a = a^3/4b + b/8c^2 + 1/4a + 1/4a + c/2a. For
positive reals AM >= GM and the equality occurs when all the numbers are equal.
Thus, a^3/4b + b/8c^2 + 1/4a + 1/4a + c/2a >=
5(a^3/4b*b/8c^2*1/4a*1/4a*c/2a)^1/5 = 5/4. Thus, p = 5/4 when a^3/4b =
b/8c^2 = 1/4a = c/2a.
(II) a+b=8-2c; a^2+b^2 >= 1/2*(a+b)^2 = 1/2(8-2c)^2 => 25-2c^2 = 1/2(82c)^2 => 1/2 <= c <= 7/2. Thus, q is 7/2 when a=b.
Hence, choice (b) is the correct option
Type: When product of terms needs maximization and sum is
constant
The sum of real numbers x and y is 1. The maximum value of xy(x^3 + y^3) is
xy(x^3 + y^3) = xy(1-3xy) = 1/3*(3xy*(1-3xy)). Thus, 3xy + (1-3xy) is constant =>
max of 3xy*(1-3xy) occurs when 3xy = 1-3xy => xy = 1/6 => p = 1/12

Type: When squaring helps, or use Partial differentiation


What is the minimum value of the expression 6x2+3y2-4xy-8x+6y+2?
(a) -13/7

(b) -2

(c) -8/5

(d) -7/3

(e) none of the foregoing

Solution:
The expression 6x2+3y2-4xy-8x+6y+2 can be written as (14x2 -12x - 3)/3 + 3(y
- (4x-6)/6)2
To minimize (y - (4x-6)/6) = 0 and 14x2 -12x - 3 should be minimum.
14x2 -12x - 3 is minimum for x = 3/7 => y = -5/7.
Hence, option (a) is the right answer
Another Method
Differentiate the expression with respect to x i.e. 12x -4y 8 = 0
Differentiate the expression with respect to y i.e. 6y -4x +6 = 0
Solve these equations to get desired x and y, and put in original expression to get
our answer
Type: Max of Min, or Min of Max
For each real number , let
be the minimum of the numbers
and
. Then the maximum value of
is

Plot the graph of f(x)= 4x+1; f(x) = x+2; f(x) = -2x+4


f(x) = x+2; f(x) = -2x+4 intersect at (2/3,8/3) and f(x)= 4x+1; f(x) = x+2 intersect
at (1/3,7/3) hence max f(x) is at 8/3.
Hence, option (E) is the right answer
Type: Max product when sum is constant (Avoid Pitfall!)
If p, q are real numbers such that p2 + pq + q2 = 1, then the greatest value of the
expression (p3q+ q3p) is

(A) 1/4

(B) 2

(D) 2/5

(D) 1/3

(E) 2/9

We need to find the greatest value of pq(p^2+q^2) given p^2 + pq + q^2 = 1, but
p^2 + q^2 >= 2pq => p = q [p^2 + q^2 = pq doesnt have any real solution set].
Hence, choice (e) is the correct answer
Type: Max of Min, or Min of Max
a, b, c, d, e, f, g are non-negative such that a+b+c+d+e+f+g = 1. Then the
minimum value of max(a+b+c, b+c+d, c+d+e, d+e+f, e+f+g) is
(a) 1/3

(b) 3/7

(c) 1

(d) 0

(e) none of the foregoing

Since there are 3n-2 terms, put a=d=f=1/3


Type: Evaluating a expression in another form
If three positive real numbers x, y, z satisfy y-x = z-y and xyz = 9, then the
minimum possible value of y is
(1) 3

(2) 32/3

(3) 31/3

(4) 1

(5) none of these

Let the numbers be y-d, y, y+d => y(y^2-d^2) = 9 => y is max when d = 0 and
hence x=y=z
Type: Inequality problem in DS Format
Each question is followed by two statements X and Y. Answer each question using
the following instructions:
Choose 1 if the question can be answered by X only
Choose 2 if the question can be answered by Y only
Choose 3 if the question can be answered by either X or Y
Choose 4 if the question can be answered by both X and Y
Choose 5 if the question can be answered by neither X and Y
Let x and y be positive real numbers. Is x^2 + y^2 < 1?
(X) y^3 + y <= x - x^3
(Y) |x| + |y| < 2 and |x| < 1 and |y| < 1

Solve yourself, Answer is option (3)


Type: Inequality problem through substitution and symmetry
Let x and y be positive real numbers such that x^3 + y^3 = 4x^2. A is the
maximum value of x + y.
Let a, b, c be real such that a+b+c = 5 and ab + bc + ca = 3. B is the largest
possible value of c.
Then A + B lies in the range
(1) [7, 8) (2) [8, 9) (3) [9, 10) (4) [10, 11) (5) none of these
Put, y = kx for 1st problem => x^3 + k^3x^3 = 4x^2 => x(1+k^3) = 4
We need to maximize x + y = x + kx = x (1+k) = 4(1+k)/(1+k^3) = 4/(k^2 k+1)
= 4/((k-1/2)^2 + ) = 16/3 = A.
2nd problem is CAT 2002 problem, solve yourself. B = 13/3
(Hint, put a = b)
Type: Inequality problem of Maximizing product when sum is
constant
Let -2 < x < 3, 0 < y < 4, 2 < z < 5. If (3-x)(4-y)(5-z)(3x+4y+5z) achieves the maximum
possible value then which among the following is not true?
(a) 3x+4y = 0

(b) |x| < |y|

(c) z = 5/2

(d) none of the foregoing

(3-x)(4-y)(5-z)(3x+4y+5z) = 1/60*(9-3x)(16-4y)(25-5z)(3x+4y+5z). Take 9-3x =


A, 16-4y = B, 25-5z = C, 3x+4y+5z = D. A+B+C+D = 50 => ABCD is max when
A=B=C=D. Solving we get x = -7/6, y = 7/8, z = 5/2.
Hence, choice (b) is the right answer

Type: Inequality problem of Maximizing product when sum is


constant
Let p, q be positive real numbers such that p + 2q = 1.
Then max value of p^3.q is
We need to maximize 27/2(p/3.p/3.p/3.2q) given p/3 + p/3 + p/3 + 2q = 1
p/3 = 2q => q = 1/8, and p = .

Type: Inequality problem of Minimizing sum when product is


constant
Let p, q be positive real numbers such that p^3.q = 1.
Then min value of p+2q is
p + 2q = p + 2/p^3 = p/3 + p/3 + p/3 + 2/p^3 >= 4(2/27)^1/4 [from AM_GM]

Вам также может понравиться